if 4lm+3=18 then m =​

Answers

Answer 1

Answer:

4(m+3)=18

m+3=18/4=9/2

m=9/2-3

m=9-6/3

m=3/3=1

so,

m=1

Step-by-step explanation:


Related Questions

Consider the graph below: Point T(-2; 3) is a point on the Cartesian Plane such that B is the angle of inclination of OT. T(-2;3) у х 2.1 Calculate the following without the use of a calculator: a) tanſ b) 13 sin B.cosB (2)​

Answers

Answer:

(a) - 3/2

(b) - 78/25

Step-by-step explanation:

According to the trigonometry, the tangent of any angle is the ratio of rise to the run of the right angle triangle .

The sine of an angle is the ratio of rise to the hypotenuse of the right angle triangle.

The cosine of an angle is the ratio of run to the hypotenuse of the right angle triangle.

(a)

[tex]tan\beta = \frac{3}{-2} = \frac{-3}{2}[/tex]

(b)

[tex]13 sin\beta cos \beta = 13\times \frac{3}{\sqrt{3^2+2^2}}\times\frac{-2}{\sqrt{3^2+2^2}}\\\\13 sin\beta cos\beta = \frac{- 78}{25}[/tex]

25 POINTS!!!!!!
Which is true about the solution to the system of inequalities shown? y<1/3x-1

Answers

Answer:

All values that satisfy [tex]y[/tex] ≤ [tex]\frac{1}{3} x-3[/tex] are solutions

Step-by-step explanation:

The reason why the other equations solutions aren't solutions are because it doesn't satisfy the second equation, but the second equation satisfy both equations because the solutions of the second equations will be in both equations.

Hope this helps

Which of the following theorems verifies that DEF = XYZ?

Answers

Answer:

AA

Step-by-step explanation:

HL = requires actual measurements of sides

LL = There are no measurements for the legs

HL = There are no measurements for the legs or hypotenuse

HA = Doesn't exist

What is the value of (-3 + 31) + (-2+31)?

Answers

Answer:

57

Step-by-step explanation:

31-3=28

31-2=29

28+29=57

-3+31-2+31= 57

Best of wishes!!

A.54 pie cm^3

B.72 pie cm^3

C.126 pie cm^3

D.378 pie cm^3

Answers

Answer:  54pi  cm^3   (choice A)

==========================================================

Explanation:

The radius of each sphere is r = 3

The volume of one sphere is

V = (4/3)*pi*r^3

V = (4/3)*pi*3^3

V = 36pi

That's the volume of one sphere.

Three spheres take up 3*36pi = 108pi cm^3 of space.

---------------------------

The radius of the cylinder is also r = 3, since each tennis ball fits perfectly in the container.

The height is h = 18 because we have each ball with a diameter 6, which leads to the three of them stacking to 3*6 = 18.

The volume of the cylinder is...

V = pi*r^2*h

V = pi*3^2*18

V = 162pi

-------------------------

Subtract the volume of the cylinder and the combined volume of the spheres:  162pi - 108pi = (162-108)pi = 54pi

This is the exact volume of empty space inside the can.

This points to choice A as the final answer

solve for surface area

formula for cube:

SA = 2 (s times s) + (4s) ( H)

Answers

Answer: 4ft^2
Please mark brainliest

(View attachment)

a) Write ordered pairs.
b) Write the domain and range.
c) Why isn't the relation a function?
d) Which ordered pair should be removed to make the relation a function?

Answers

Answer:

in a relationship that maps elements from one set (the inputs) into elements from another set (the outputs), the usual notation for the ordered pairs is:

(x, y), where x is the input and y is the output.

In this case, the point where the arrow starts is the input, and where the arrow ends is the output.

a)

The ordered pairs are:

(28, 93)

(17, 126)

(52, 187)

(34, 108)

(34, 187)

b) The domain is the set of the inputs, in this case the domain is the set where all the arrows start, then the domain is:

{17, 28, 34, 52}

And the range is the set of the outputs, in this case the range is:

{93, 108, 126, 187}

c) A function is a relationship where the elements from the domain, the inputs, can be mapped into only one element from the range.

In this case, we can see that the input {34} is being mapped into two different outputs, then this is not a function.

d) We can remove one of the two ordered pairs where the input is {34},

So for example, we could remove:

(34, 108)

And then the relation would be a function.

please help me please help me please help me please help me please help me please help me please​

Answers

Answer:

Q3. 9

Q4. 6

Step-by-step explanation:

if the bookstore pays $60 to the publisher what will be the selling price?

Answers

Answer:

This means that, when the price of a book from a publisher is $60, the bookstore will sell it for $88 to the students.

If f(x) = - 2x +5 and g(x)=x2-1, then f(-3)+g(2) =​

Answers

Answer:

[tex]{ \tt{f(x) = - 2x + 5}} \\ { \boxed{ \bf{f( - 3) = - 2( - 3) + 5 = 11}}} \\ \\ { \tt{g(x) = {x}^{2} - 1}} \\ { \boxed{ \bf{g(2) = {2}^{2} - 1 = 3}}} \\ f( - 3) + g(2) = 11 + 3 \\ = 14[/tex]

Is it true or false that for all sets A, B, and C, A U (B - C) = (A U B) - C?

Answers

Answer:yes

Step-by-step explanation:66

The given statement  A U (B - C) = (A U B) - C is true.

What is a set ?

A set is collection of well defined objects.

According to the given question we have to state whether A ∪ ( B - C ) = ( A ∪ B ) - C.

Lets consider we have three sets A, B and C and we also consider they intersect each other.

( B - C ) represents the elements which belongs to B but not in C.

∴ A ∪ ( B - C ) represents the no. of elements which belongs to the set B but not in C union the no. of elements which belongs to A.

AND

( A ∪ B ) - C represents no. of elements which belongs to A or B but not in C.

learn more about sets here :

https://brainly.com/question/8053622

#SPJ2

Find tan 0, where is the angle shown. Give an exact value, not a decimal approximation. (PLZ HELP DUE SOON I GIVE BRAINLIST :D)​

Answers

Answer:

[tex]\frac{24}{7}[/tex]

Step-by-step explanation:

Tanθ=Opposite/Adjacent

we have the adjacent side but need the oppsoite

We will use a²+b²=c²

25²=7²+b²

576=b²

b=24

Therefore the answer is

[tex]\frac{24}{7}[/tex]

Find an equation for the line parallel to 3x-5y=2 with​ y-intercept (0,1/5). Write the answer in ​ slope-intercept form.

Answers

y= 3x/5 - 2/5 .i think

what percentage of 7 1/2 is 2 1/2

Answers

Answer:

7+½ = (7*2+(1))/2=15/2

Step-by-step explanation:

2+½ =5/2 and. (15/2)/(5/2)=3. this means %33.3333

3 3/4 × 2 2/9 please
Help ‍♀️‍♀️‍♀️

Answers

[tex]\implies {\blue {\boxed {\boxed {\purple {\sf { \: 8 \frac{1}{3}\:(or) \:8.333}}}}}}[/tex]

[tex]\sf \bf {\boxed {\mathbb {Step-by-step\:explanation:}}}[/tex]

[tex]3 \frac{3}{4} \times 2 \frac{2}{9} [/tex]

➺[tex] \: \frac{15}{4} \times \frac{20}{9} [/tex]

➺[tex] \: \frac{300}{36} [/tex]

➺[tex] \: \frac{25}{3} [/tex]

➺[tex] \: 8 \frac{1}{3} [/tex]

➺[tex] \: 8.333[/tex]

[tex]\large\mathfrak{{\pmb{\underline{\pink{Mystique35 }}{\orange{❦}}}}}[/tex]

which equation is the inverse of 5y+4=(×+3)^2+1/2?​

Answers

Answer:

The inverse is -3 ±sqrt(5x+7/2)

Step-by-step explanation:

5y+4=(x+3)^2+1/2?

To find the inverse, exchange x and y

5x+4=(y+3)^2+1/2​

Solve for y

Subtract 1/2

5x+4 -1/2=(y+3)^2+1/2​-1/2

5x+8/2 -1/2=(y+3)^2+1/2​-1/2

5x+7/2 = (y+3)^2​

Take the square root of each side

±sqrt(5x+7/2) =sqrt( (y+3)^2​)

±sqrt(5x+7/2) = (y+3)

Subtract 3 from each side​

-3 ±sqrt(5x+7/2) = y+3-3

-3 ±sqrt(5x+7/2) = y

The inverse is -3 ±sqrt(5x+7/2)

Gasoline sells for 1.3 euros per liter. What is the price in US dollars per gallon? (recall that 1 gal = 3.785 L)

British pound is 1.212 to 0.8251 USD

Answers

4.67 $/gal is the price of gasoline.

Step-by-step explanation:

Given:

Price of gasoline = 1.3 €/L

1 gallon is equals to 3.785 Liters

1 euros is equals to  0.9497 US dollars

To find:

The price of gasoline in US dollars per gallon

Solution:

Price of the gasoline = 1.3 €/L

[tex]1 gal = 3.785 L\\1L=\frac{1}{3.785} gal\\ 1.3 euro /L=\frac{1.3 euro }{\frac{1}{3.785 }gal}\\=\frac{1.3 euro \times 3.785 }{1 gal}=4.9205 euro /gal[/tex]

Now convert euros to US dollars by using :

1 euros = 0.9497 $

The price of gasoline in US dollar per gallons:

[tex]4.9205 euro/gal=4.9205 \times 0.9497 \$/gal\\=4.6730 \$/gal\approx 4.67 \$/gal[/tex]

4.67 $/gal is the price of gasoline.

Learn more about conversions:

brainly.com/question/13076223

brainly.com/question/2904463?referrer=searchResults

Help find instantaneous rate of change :)!

Answers

Answer:  0

=========================================================

Explanation:

Let's say that point A is at (0,0) and B is somewhere else on the parabola.

I'll make point B go to the right of point A.

For now, let's say B is at (4,16).

If we compute the slope of line AB, then we find the average rate of change (AROC). The AROC in this case is (y2-y1)/(x2-x1) = (16-0)/(4-0) = 16/4 = 4. Because point A is at (0,0), we're really just computing y/x where the x,y values come directly from point B.

--------------

Now let's move B to (3,9). If we used the slope formula again, we would get the slope of 3. Note how y/x = 9/3 = 3.

Then let's move B to (2,4). The AROC is now y/x = 4/2 = 2

As B gets closer to A, the AROC is decreasing. The AROC is slowly approaching the IROC (instantaneous rate of change).

--------------

Point B is generally located at (x,x^2) for any real number x. Keeping A always fixed at the origin, the slope of line AB is y/x = (x^2)/x = x.

What does this all mean? It means that if x = 0, then the IROC is 0. You might be quick to notice that we cannot divide by zero. So instead of letting x be zero itself, we'll just get closer and closer to it. This is where the concept of limits come into use. This is what calculus is based on (both integral and differential calculus).

Anyway, when calculating the IROC, we're really calculating the slope of the tangent line to the f(x) curve. Refer to the diagram below.

----------------

In short, the slope of the tangent line at x = 0 is m = 0. We have a flat horizontal line that touches the parabola at (0,0).

please help meeeee!!​

Answers

Step-by-step explanation:

[tex]\begin{aligned} -5x+4y &= 3\\\\ x&=2y-15 \end{aligned}[/tex]

Paige and her family went to the movies. They bought 4 tickets and paid $12 for popcorn. They spent $40. How much did each ticket cost?

I need equation and cost :)

Answers

Answer:

Cost of tickets: $7. Equation: 40 = 4x + 12.

Step-by-step explanation:

Answer:

4*t +12 = 40

Each ticket cost 7 dollars

Step-by-step explanation:

tickets + popcorn = total cost

4*t +12 = 40

Subtract 12 from each side

4t +12-12 = 40-12

4t = 28

Divide by 4

4t/4 = 28/4

t = 7

Each ticket cost 7 dollars

please help show steps thx

Answers

Answer:

1) P = 282m, A = 141m^2

2) P = 82.4in, A = 40in^2

3) P = 62.7m, A = 73.1m^2

Step-by-step explanation:

1) top:L=12m, W=3m, mid: l= 12m, w= 12-7 = 5m, bot: l=15m, w=3m

Perimeter= 2(lw)

P = 2(12x3) + 2(12x5) + 2(15x3)

P = 2(36) + 2(60) + 2(45)

P = 72 + 120 + 90

P = 282m  

Area= lw

A = (12x3) + (12x5) + (15x3)

A = 36 + 60 + 45

A = 141m^2

2) Rectangle:l=7in, w=5in, Right Triangle:a=5-3=2in, b=12-7=5in

Perimeter= 2(lw) + (a+b+sqrt(a^2+b^2))

P = 2(7x5) + (2+5+sqrt(2^2+5^2))

P = 70 + 12.39

P = 82.4in

Area= lw + ((ab)/2)

A = (7x5) + ((2x5)/2)

A = 35 + 5

A = 40in^2

3) Semi-Circle: d=8m, r=8/2=4m, Right Triangle: a=8m, b=12m

Perimeter= pid + (a+b+sqrt(a^2+b^2))

P = 9pi + (8+12+sqrt(8^2+12^2))

P = 28.27 + 34.42

P = 62.7m

Area= (1/2)pir^2 + ((ab)/2)

A = (1/2)pir^2 + ((ab)/2)

A = (1/2)pi(4)^2 + ((8x12)/2)

A = 25.13 + 48

A = 73.1m^2

differentiate loge(x/x^2+7)​

Answers

Answer:

1+1=11 2+2=22 ok na yan kuya or ate

Which of the following equations is NOT an example of inverse variation?
I. f(x)=kx
II. f(x)=2kx
f
(
x
)
=
2
k
x
III. f(x)=−kxz

Answers

Based on the equation,the equation which is not an inverse variation is f(x) = kx

Inverse variation

f(x) = kx

= k × x

f(x) = 2k/x

= k(2/x)

f(x) = -kx/z

= k(-x/z)

Inverse variation is given by:

y = k(1/x)

where,

k = constant of proportionality

Therefore, f(x) = kx is not an inverse variation.

Learn more about inverse variation:

https://brainly.com/question/13998680

#SPJ1

I NEEEEED HELP!!!!!!

Answers

can you take a better photo?

fill in the blanks the 2 digit largest whole number is______​

Answers

99 is the correct answer

Assume that Z has a standard normal distribution. Determine the value for z that solves each of the following.

a. P(-z < Z < z) = 0.95 (Round your answer to two decimal places (e.g. 98.76))
b. P(-z < Z < z) = 0.99 (Round your answer to two decimal places (e.g. 98.76))
c. P(-z < Z < z) = 0.68 (Round your answer to three decimal places (e.g. 98.765))
d. P(-z < Z < z) = 0.9973 (Round your answer to two decimal places (e.g. 98.76))

Answers

Answer:

a)  P  ( - 1.96  < Z < 1.96 )

b) P  ( - 2.58 < Z < 2.58)

c) P (  -0.995  < Z < 0.995 )

d) P  ( - z  <  Z  < z )     =  P ( ( Z ± 3σ )   then that is close to 1

Step-by-step explanation:

a)  P  ( - z  <  Z  < z )     =   P  ( - 1.96  < Z < 1.96 )

CI = 95 %   significance level  α  = 5 %   α  = 0.05    α/2 = 0.025

z = 1.96

b)  P  ( - z  <  Z  < z )     =   P  ( - 2.58 < Z < 2.58)

CI = 99 %   significance level  α  = 1 %   α  = 0.01    α/2 = 0.005

z = 2.58

c) P  ( - z  <  Z  < z )     =   P (  -0.995  < Z < 0.995 )

CI = 68 %   significance level  α  = 32 %   α  = 0.32    α/2 = 0.16

z ≈ 0.9954

We interpolate in this case

1             ⇒  0.1587

0.99      ⇒  0.1611

0.01    ⇒   0.0024        

 x       ⇒    0.0013       x  =  0.01 *0.0013  /  0.0024  

x =  0.005416

and    z =  0.99 + 0.005416

z = 0.9954

d) P  ( - z  <  Z  < z )     =   P (  - 0.00 < Z < 0. 00)

CI = 0.9973 %   significance level  α  = 0.0027 %   α  = 0.000027             α/2 = 0.0000135

z = 0.00003375     ⇒ z = 0.00

NOTE: The value of α  is too small. The Empirical Rule establishes that 99.7 % of all values in a normal distribution fall in the interval ( Z ± 3σ)

that means all the values. Then the probability of finding the random variable between that range is close to 1 and we can not find in tables a number to approximate just with only two decimal places

Help me outtttttttttto

Answers

Answer:

,

Step-by-step explanation:

hear is your answer please give me Some thanks

PLEASE HELP!

Determine which of the following lists is in order from smallest to largest.

1. -3,131,0, (-3)^2
2. (-3)^2,-3,0, |3|
3. -3,0,|3|, (-3)^2
4. 0,-3,|3|, (-3)^2

Answers

Answer:

3. -3,0,|3|, (-3)^2

Step-by-step explanation:

Answer:

answer would be option 3

Step-by-step explanation:

help this helps

Find the difference quotient of f; that is, find f(x+h)-f(x) , h ≠ 0 for the following function

f (x)=6x+8

f(x+h)-f (x)/h =

Answers

See56.7 cause it is the correct one to the 20th

The sides of a square field are 24 meters. A sprinkler in the center of the field sprays a circular area with a diameter that corresponds to a side of the field. How much of the field is not reached by the sprinkler? Round your answer to the nearest hundredth. Use 3.14 for π.

Answers

Answer:

A

Step-by-step explanation:

i took the test

Other Questions
What does It in line 6 refer to? Support your answer with evidence from the poem. yo brainly bolws and 2+2 is 4 Which form of poetry is used in TS Eliots preludes fixed form. Closed form. Free verse. Or blank verse What is the volume of the following cylinder? Do not round your answer. (Use 3.14 for .)volume = ____yd3 Karen purchased 3 gallons of yellow paint and 4 gallons on blue paint from the hardware store. The total cost was $105. Yellowpaint and blue paint sell for the same price per gallon. Which THREE statements are correct? please help me please Why was there an intense rivalry among the trading companies of different European Companies Name the marked angle in 2 different ways. When did Asoka start following Buddhist values? We didn't enjoy our food when we went out to eat, but our friend Nelson said that the restaurant normally served some of the best Italian food in town. In fact, he said it was one of his favorite places to eat. He said that we should return and give it a second chance. What word best replaces "said" in the last sentence? insisted announced exclaimed shouted Find the GCF of 20 and 95 Using the Euclidean Algorithm. Check your work by using the prime factorization method. Help please ;-------; write a couple of paragraph writing your plans and intention that you are going to when you will be meet your teachers and friends physically https://hegartymaths.com/assessmentSeveral points are given on the graph below.a) A, B and C are three corners of a rectangle.Write the coordinates of the other corner.b) C, D and E are three vertices of a rhombus.Write the coordinates of the other vertex.5 ty..4.321.-5-4-3-1234-2E5-1D.-2-3-4-5 What is dopamine?? 1.)feel good hormone 2.) a drug teens use 2.) a form of a pain killer 3.)type of marijuana As a manager, you are trying to decide how to best layoff 10% of your workforce. You are considering laying off the 10% who are the oldest in age, because you feel that they are closest to retirement anyway. Which of the following types of justice would you most likely violate the rules of by doing this?A. DistributiveB. ProceduralC. InterpersonalD. Informational A grapefruit is 8% heavier than an orange, and an apple is 10% lighter than theorange.By what percentage is the grapefruit heavier than the apple?pls answer What is the value of log Subscript 5 Baseline 125? A) m + nB) 0C) m - 1D) -(m+n) Help!! Stuck on math problem!Fast answers are appreciated, thanks :)The graph of the function has a vertical asymptote of x = ______The graph of the function has a horizontal asymptote of y = ______